8/24-25-Mixed-subject MBE PQs 2

Lakukan tugas rumah & ujian kamu dengan baik sekarang menggunakan Quizwiz!

Question 6245 A doctor walked through the front door of a police station and asked to see the police chief. When the chief appeared, the doctor told him that he wanted to confess to murdering his wife. Before talking with him further, the chief read the doctor his Miranda rights. He asked the doctor to sign a document stating that he understood his Miranda rights and waived them. The doctor complied with the request. The doctor then said, "I killed her with the fish knife she kept in the wood block on the counter. I stabbed her in the heart and severed an artery. She is lying in our kitchen." At that very moment, police officers were at the doctor's house, having received a 911 call from a neighbor. They found the dead wife. Three weeks later, the court appointed a psychiatrist to examine the doctor. The psychiatrist reported that the doctor was mentally ill, could not make rational choices, could not understand his rights, and was compelled by his illness to confess. Is the doctor's confession admissible? Answers: Yes, because the police did not coerce the doctor into confessing. Yes, because there was no custodial interrogation. No, because the defendant was diagnosed as mentally ill by a medical professional. No, because a mentally ill person cannot waive his Miranda rights.

Answer choice A is correct. A confession must be voluntary to be admissible. Here, the doctor's confession was voluntary, and the police did not coerce him into making it. Answer choice B is incorrect. Because the doctor had been given Miranda warnings prior to making his confession, the fact that he was neither in police custody nor subject to police interrogation is irrelevant. Either of these facts would excuse the failure of the police to give Miranda warnings to the doctor prior to the doctor making his statement. However, the issue here is whether the doctor's confession was a product of his own free will in the absence of a custodial interrogation. Answer choice C is incorrect because, although the defendant's age, state of health, education, or intoxication are all factors in determining the coercive nature of the confession, the defendant's mental condition alone cannot violate the voluntariness standard. There must be coercive police activity for the confession to be found involuntary. In this case, there was no coercive conduct by the police, so the confession was voluntary. Answer choice D is incorrect because the Miranda waiver is irrelevant; volunteered statements are not protected by Miranda, as they are, by definition, not the product of interrogation.

Question 5019 Three brothers owned several luxury vacation rental properties in the Caribbean. The brothers were organized as a limited partnership, which was registered in State B. The two brothers who handled the day-to-day business were the general partners, and they lived close to one another in State A. The third brother, a resident of State B, was the limited partner whose role was limited solely to providing capital. The plaintiff, a resident of State B, rented a property from the brothers, where she intended to stay for the winter. She prepaid in full for the entire five-month stay. Upon her arrival, she found that the property had been completely destroyed by a hurricane. The plaintiff brought a breach of contract suit in federal district court naming the partnership as defendant. She seeks $80,000 in damages. If the limited partnership moves to dismiss for lack of jurisdiction, will it prevail? Answers: Yes, because the limited partner resides in State B. Yes, because the limited partnership is registered in State B. No, because the partner who resides in State B is merely a limited partner. No, because the majority of the partners reside in State A.

Answer choice A is correct. A partnership is considered a citizen of each state in which each of its partners is domiciled. This is true for both general and limited partnerships; a limited partnership is a citizen of every state in which its general and limited partners are domiciled. Here, the limited partner resides in State B. Because the plaintiff also resides in State B, complete diversity does not exist and the federal district court does not have jurisdiction. Answer choice B is incorrect. A limited partnership is a citizen of each state in which each of its partners are domiciled, not the state where it is registered. Answer choice C is incorrect. The limited partnership is a citizen of each state where each general partner and each limited partner resides. Answer choice D is incorrect. The limited partnership is a citizen of each state where each general partner and limited partner resides, not where the majority of the partners reside.

Question 5837 In a trial for civil assault, the defendant called his friend to testify regarding the defendant's location at the time of the assault. The friend unexpectedly testified that the defendant was not with the friend and that he was unsure where the defendant was at the time in question. At his deposition a month before, the friend had testified that he was with the defendant when the assault occurred. The defendant then sought to introduce the deposition transcript of his friend's testimony. Is the deposition transcript admissible? Answers: Yes, both to impeach the friend and as proof of the defendant's location at the time of the assault. Yes, but only to question the truthfulness of the friend's trial testimony. No, because a party may not impeach its own witness. No, because the statement was not made at a prior trial.

Answer choice A is correct. A prior inconsistent statement made under penalty of perjury at a trial, hearing, or other proceeding, or in a deposition may be admissible to impeach the declarant's credibility and as substantive evidence. In this case, the defendant sought to introduce the friend's deposition transcript as a prior inconsistent statement. Because the friend's prior inconsistent statement regarding the defendant's location at the time of the assault was made at a deposition, the statement may be admitted both to impeach the friend and as substantive evidence of the defendant's location. Answer choice B is incorrect. While the friend's statement in the deposition transcript may be used to impeach the friend's testimony at trial, the statement may also be admitted as substantive evidence. Answer choice C is incorrect because any party, including the party that called the witness to testify, may impeach a witness. Answer choice D is incorrect. Because the friend's statement was made at a deposition, which is an event that requires the deponent to answer questions under penalty of perjury, this statement is not hearsay and is admissible.

Question 8388 Seeking to prevent misleading advertisements, a state law permits dentists to advertise that they are specialists only in areas recognized by the American Dental Association (ADA) as specialties. A dentist whose practice is limited to implant dentistry, an area of specialty that is not recognized by the ADA, has challenged the law as a violation of her First Amendment Free Speech Rights as made applicable to the states through the Fourteenth Amendment. Of the following, which would be best argument that the state can advance for upholding this law? Answers: An advertisement by a dentist as a specialist in a non-ADA recognized area is inherently misleading. Commercial speech is not protected by the First Amendment. The law satisfies the strict scrutiny test. The state has a substantial interest in protecting consumers.

Answer choice A is correct. Commercial speech (i.e., advertising and similarly economically oriented expression) is entitled to an intermediate level of First Amendment protection. Restrictions on commercial speech are reviewed under a four-part test. First, the commercial speech must concern lawful activity and be neither false nor misleading. Second, the asserted governmental interest must be substantial. Third, the regulation must directly advance the asserted interest. Fourth and finally, the regulation must be narrowly tailored to serve that interest; this means there must be a "reasonable fit" between the government's ends and the means chosen to accomplish those ends. Here, if an advertisement that a dentist is a specialist in a non-ADA recognized area of practice is inherently misleading, the speech is unprotected commercial speech. Answer choice B is incorrect because commercial speech is entitled to an intermediate level of First Amendment protection. Answer choice C is incorrect because commercial speech is entitled to an intermediate level of First Amendment protection. Answer choice D is incorrect because, while the existence of a substantial state interest would satisfy one part of the four-part test for state regulation of commercial speech, a finding that the prohibited advertisements would be inherently misleading would determinatively establish that the speech is unprotected.

Question 5928 A man broke into his ex-wife's house one night, thinking it was unoccupied, with the intent to steal jewelry given to her by her new boyfriend. He carried a revolver, "just in case." Far from being unoccupied, the living room into which the man entered was filled with people waiting in the dark to throw a surprise party for his ex-wife's new boyfriend. The ex-wife, startled and frightened, ordered the man to leave immediately. Infuriated that his ex-wife had a new boyfriend, the man removed the revolver and fired it wildly over his ex-wife's head, intending to scare her. One of the bullets, ricocheting off a ceiling light fixture, struck and killed one of the man's closest friends. The jurisdiction has abolished the felony murder rule, but otherwise follows the common law of murder. The crimes below are listed in descending order of seriousness. On these facts, what is the most serious offense of which the man properly could be convicted? Answers: Murder. Voluntary manslaughter. Involuntary manslaughter. Burglary.

Answer choice A is correct. Even though he lacked the specific intent to kill and the jurisdiction does not recognize felony murder, the man could properly be convicted of murder because he could be found to have acted with a depraved heart. A killing that results from conduct by a defendant that manifests a reckless indifference to an unjustifiably high risk to human life is a depraved-heart murder. In this case, the man's reckless act of wildly firing his gun over his ex-wife's head in a room filled with people manifested a reckless indifference to human life. Answer choice B is incorrect because the man could not properly be convicted of voluntary manslaughter because he lacked the requisite intent to kill and there was no adequate provocation for his conduct. This answer is also incorrect because he could be convicted of the more serious offense of depraved-heart murder. Answer choice C is incorrect because, although the man could properly be convicted of involuntary manslaughter, he could also be convicted of the more serious offense of depraved-heart murder. Answer choice D is incorrect. Although the man may properly be convicted of burglary, he could also be convicted of the more serious offense of depraved-heart murder.

Question 8435 A tire dealer phoned in an order to a wholesaler for various tires of specific sizes and types at a total cost of $25,000. The wholesaler sent the dealer an acceptance of the order that listed the quantities of the various sizes and types of tires that the dealer had ordered. The acceptance identified the wholesaler in its heading, and in its body it identified the dealer as the person who had ordered the tires. Twelve days after the dealer received the acceptance but prior to the wholesaler's shipment of the tires, the dealer objected to the order. Unable to resell the tires for $25,000, the wholesaler sued the dealer for damages. The dealer asserted the Statute of Frauds as a defense. Will the dealer be successful in its defense against the wholesaler's action? Answers: No, because the dealer failed to timely object to the written acceptance. No, because there is no need for a writing when both parties to a contract for the sale of goods are merchants. Yes, because the dealer never signed a writing that identified it as the buyer of the tires. Yes, because the dealer objected to the written acceptance before the wholesaler had shipped the tires.

Answer choice A is correct. If both parties are merchants and a memorandum sufficient against one party is sent to the other party, who has reason to know its contents, and the receiving party does not object in writing within 10 days, then the contract is enforceable against the receiving party even though he has not signed it. Here, the dealer did not object to the written acceptance sent by the wholesaler within 10 days. Therefore, the contract as expressed in the written acceptance is enforceable against the dealer despite the absence of the dealer's signature. Answer choice B is incorrect because the UCC does not completely eliminate the need for a writing when both parties are merchants. Answer choice C is incorrect. Generally, the Statute of Frauds requires that there be a writing signed by the party against whom the contract is to be enforced. However, there is an exception when both parties are merchants if a memorandum sufficient against one party is sent to the other party, who has reason to know of its contents, and the receiving party does not object in writing within 10 days. Answer choice D is incorrect. Under the special memorandum provision of the UCC Statute of Frauds that is applicable when both parties are merchants, a party who receives a memorandum of the contract from the other party has 10 days to object in writing to the memorandum, or the party receiving the memorandum waives the Statute of Frauds defense to the contract.

Question 7427 A state statute permits a tax credit for contributions made to school tuition organizations. These organizations provide scholarships for students to attend private schools, including religious schools, within the state. In a federal district court, a plaintiff, as a taxpayer, has challenged the statute as a violation of the Establishment Clause of the First Amendment to the U.S. Constitution. Does the plaintiff have standing to bring this lawsuit? Answers: No, because the plaintiff does not have standing to bring this action as a taxpayer. No, because the statute in question is a state rather than federal statute. Yes, because the plaintiff cannot claim this tax credit without contributing to a school tuition organization that provides scholarships for students to attend religious schools. Yes, because the plaintiff's challenge is based on a violation of the Establishment Clause.

Answer choice A is correct. In general, a taxpayer does not have standing to file a federal lawsuit simply because the taxpayer believes that the government has allocated funds in an improper way. While there is a very narrow exception for a taxpayer suit challenging a specific legislative appropriation made under taxing and spending powers for violation of the Establishment Clause, this exception does not apply when the government provides a benefit in the form of a tax credit rather than an appropriation. Answer choice B is incorrect because a taxpayer has standing to challenge a state as well as a federal legislative appropriation as a violation of the Establishment Clause of the First Amendment to the U.S. Constitution. Answer choice C is incorrect because the mere unavailability of a certain tax credit to an individual taxpayer does not give that taxpayer standing to challenge the constitutionality of the tax credit. Answer choice D is incorrect because, while there is a narrow exception to the general rule prohibiting taxpayer standing to challenge violations of the Establishment Clause, the exception does not apply when the government provides a benefit in the form of a tax credit, rather than an appropriation.

Question 7839 An attorney representing a defendant filed an answer in which he truthfully asserted that, after a reasonable opportunity for further investigation, it was likely that the evidence would show that the statute of limitations had expired with respect to the plaintiff's claim. The attorney signed the answer and included his email address and telephone number, but did not attach an affidavit attesting to the truthfulness of the assertions in the answer. At a pretrial conference, the attorney asserted that the evidence he now had established that the statute of limitation had expired. In fact, the attorney knew that the evidence eliminated the statute of limitations as a defense. The plaintiff's attorney served a motion for sanctions under Rule 11 on the defendant's attorney, but did not file the motion with the court until a month later. May the court impose sanctions under Rule 11 on the defendant's attorney? Answers: Yes, because the defendant's attorney presented the statute of limitations defense at the pretrial conference. Yes, because the defendant's attorney failed to attach an affidavit to the answer attesting to the truthfulness of the assertions in the answer. No, because the defendant's attorney truthfully asserted the statute of limitations defense in the answer which he signed. No, because the plaintiff's attorney failed to timely file its sanctions motion with the court.

Answer choice A is correct. Rule 11 requires an attorney who is representing a client to sign every pleading, written motion, and other paper filed with the court. By presenting the paper to the court, the attorney is certifying that, among other things, the factual contentions have evidentiary support or, if specifically so identified, will likely have evidentiary support after a reasonable opportunity for further investigation or discovery. "Presenting" includes not only signing, filing, and submitting the paper, but also later advocating a position presented in the pleading. Here, at the pretrial conference, the defendant's attorney advocated the statute of limitations defense, even after he had learned that it was not supported by the evidence. Consequently, the court may impose sanctions under Rule 11 on the defendant's attorney. Answer choice B is incorrect. By signing a pleading, motion, or other paper, an attorney is certifying, to the best of her knowledge, information, and belief, formed after an inquiry reasonable under the circumstances, the existence of evidentiary support for her factual contentions or the likelihood of such support. There is no requirement that a pleading be accompanied by an affidavit attesting to the truthfulness of the matters asserted in the pleading. Answer choice C is incorrect because, while the primary focus of Rule 11 is on the filing of a pleading or other paper with the court, it also extends to later assertions of positions presented in the pleading or other paper. Answer choice D is incorrect because a party who has served a Rule 11 motion for sanctions on another party must give that party 21 days to withdraw or correct matters before filing the motion with the court.

Question 3159 A widower owned a car repair business, including the garage out of which it operated. Upon his retirement, the widower sold the business to an employee and deeded a 50 percent ownership interest in the garage to the employee in exchange for the employee's payment to the widower of the fair market value of both the business and the share of the garage. Within the same deed, the widower deeded the remaining 50 percent interest in the garage to his daughter. The deed specified that his daughter held her interest jointly with the employee's 50 percent interest and that the survivor would take sole ownership of the garage. Upon the employee's death, his real property interests passed by will to his child by a prior marriage. Who owns the garage? Answers: The daughter, because she owned the property as a joint tenant with the right of survivorship. The daughter and the employee's child each hold a 50 percent interest, because rights of survivorship may only be created between married persons. The employee's child, because the employee paid fair market value for the business while the daughter did not. The daughter and the employee's child each hold a 50 percent interest, because they are tenants in common.

Answer choice A is correct. The widower transferred to both his employee and his daughter a 50 percent interest in the garage, and each received his or her interest at the same time and in the same document. (Remember that the four unities of creating joint tenancy are unity of possession, interest, time, and title. If only the unity of possession occurs, a tenancy in common has been established, and there is no right of survivorship.) Consequently, the employee and the widower's daughter were joint tenants. The employee could not transfer by will his 50 percent interest in the garage to his child by a prior marriage because that interest automatically went to the widower's daughter upon the employee's death. Answer choice B is incorrect because parties do not have to be married to hold property as joint tenants with rights of survivorship; parties must be married in order to hold property in a tenancy by the entirety. Answer choice C is incorrect because despite the employee's payment for the business, each still received the same interest in the garage from the widower. Answer choice D is incorrect because the employee's interest went to the widower's daughter upon the employee's death. They were joint tenants rather than tenants in common.

Question 7219 A tire manufacturer sold 100 car tires to a retailer. The retailer sold four of these tires to a woman. The woman took the tires to her mechanic to install on her car, but the mechanic informed her that they were the wrong size. The woman immediately returned the tires to the retailer. The retailer examined the tires thoroughly and determined that they were in the same condition they had been in when they left the retailers store. The retailer marked the tires as "Not Brand New" and discounted them by 15%. A landscaper bought the tires at the discounted price and had them installed on the truck he used to transport his landscaping tools for his business. Three days after the tires were installed and before he had driven the truck with the new tires, he woke up to discover that all of the tires had gone flat. Due to the flat tires, the landscaper missed two landscaping appointments he had scheduled for that day. Upon investigation, it was revealed that the tires had been improperly tempered when they were manufactured. The landscaper sued the manufacturer based on strict products liability theory to recover damages for his lost business. Which of the following is the manufacturer's best defense? Answers: By paying the discounted price, the landscaper assumed the risk that the tires were not reasonably fit for the ordinary purposes for which they were being sold. The manufacturer is not liable for the landscaper's purely economic loss. The retailer's sale of the tires to the woman cut off the chain of liability to the manufacturer. There were no implied warranties on the tires because they were labeled "Not Brand New."

Answer choice B is correct. A claim for purely economic loss (e.g., loss of business) is not allowed under a strict products liability theory. Therefore, this is the manufacturer's strongest defense against a strict products liability claim by the landscaper to recover damages for his lost business. Answer choice A is incorrect. Paying a discounted price does not waive the implied warranty of merchantability. Additionally, the landscaper is bringing a claim based on strict products liability, not on a breach of warranty. Therefore, this is not a helpful defense to the landscaper's claim. Answer choice C is incorrect because the condition of the tires did not change while they were in the woman's possession. Therefore, the manufacturer would still be a proper defendant in this action if the landscaper had suffered damages that he could recover with a strict products liability action. Answer choice D is incorrect because the landscaper is not bringing a claim for breach of warranty. Therefore, any argument that the language "Not Brand New" is a disclaimer of warranties is irrelevant.

Question 8583 At a hearing on legislation supported by the President, a senator made a slanderous statement about the President. Later the same day, the senator, during an interview with a reporter, repeated the statement, seeking to motivate citizens to contact their congresspersons to encourage them to vote against passage of the legislation. Within hours of learning of the senator's statement, the President, via social media, maligned the senator, falsely accusing her of taking bribes, and then encouraging the rest of the Senate to vote in favor of the legislation. If a civil suit for money damages is filed against either the President or the senator as a result of the statements, who may properly be held liable? Answers: Neither the President nor the senator, because neither exceeded the immunity of their offices. The senator only, because she exceeded her legislative immunity. The President only, because he exceeded his executive immunity. Both the President and the senator, because each exceeded the immunity of their offices.

Answer choice B is correct. Although members of Congress, such as senators, enjoy immunity for statements made in the regular course of the legislative process (e.g., during legislative hearings on a bill), immunity will not protect statements made outside of Congress. Accordingly, immunity will not extend to a "re-publication" of a defamatory statement, even if that statement was originally made in the Senate. On the other hand, the President enjoys absolute executive immunity to civil suits for money damages for actions while he is in office. Answer choice A is incorrect because it improperly extends legislative immunity. Answer choice C is incorrect because it both ignores executive immunity and improperly extends legislative immunity. Answer choice D is incorrect because it improperly states that the senator enjoys immunity for the republication of the defamatory statement.

Question 7244 A food critic sued a restaurant owner for damages he sustained when he allegedly contracted food poisoning at the owner's restaurant. The food critic claimed that he ate dinner at the owner's restaurant and spent the next 36 hours being so violently ill that he was hospitalized for dehydration. When the food critic was released from the hospital, he visited the restaurant owner to complain. The owner said he had no idea what happened because he was not in the restaurant the evening of the critic's meal, but in case his kitchen had indeed made the critic ill, the owner offered to pay the critic's medical bills to settle the issue out of court. The critic refused and sued the restaurant owner. At trial, the owner testified that he watched his staff prepare the critic's meal, and that he was sure all safety and health protocols had been obeyed. May the critic impeach the restaurant owner with his statement that he was not in the restaurant the evening of the critic's meal? Answers: No, because the owner did not make the statement under penalty of perjury. No, because the owner's prior statement was made during settlement negotiations. Yes, because it is a statement by a party opponent. Yes, because the statement is not being offered to prove the validity or amount of the critic's claim.

Answer choice B is correct. Compromise offers made by any party, as well as any conduct or statements made during compromise negotiations, are not admissible to prove or disprove the validity or amount of a disputed claim, nor may they be admitted for impeachment by prior inconsistent statement or contradiction. In this case, because the restaurant owner made this prior inconsistent statement during a negotiation to settle the critic's claim, it is inadmissible. Answer choice A is incorrect because a prior inconsistent statement may be admitted to impeach a witness even if it was not made under penalty of perjury. Additionally, if a public policy exception did not apply, the statement would be admissible as a statement by a party opponent (not hearsay) regardless of whether it was made under penalty of perjury. Answer choice C is incorrect. Although the owner's prior statement qualifies as a statement by a party opponent, it is nevertheless inadmissible because it was made during settlement negotiations. Answer choice D is incorrect because the public policy exception for statements and conduct during settlement negotiations also prevents the use of the statements for impeachment by prior inconsistent statement.

Question 7440 A federal law gives the President the power to cancel any limited tax benefit contained in a statute. The act requires the President to notify Congress within five days after the enactment of the cancelled benefit. The cancellation takes effect immediately, but may become null and void if Congress passes a disapproval bill by a simple majority in each house. A primary purpose of the law is to discourage Congresspersons from seeking favorable tax treatment for specific constituents within the confines of a more comprehensive tax law. Subsequently, a federal tax act was passed that contained a provision granting a tax credit that, although generically worded, applied to a single constituent of the Congressman who proposed the provision. The President, pursuant to the prior federal law, cancelled the provision granting the tax credit. Congress took no action to reinstate this credit. The recipient of this tax credit has challenged the constitutionality of the law that permitted the President to cancel this tax benefit. Of the following, which would provide the strongest constitutional basis on which to base this challenge? Answers: The Origination Clause of Article I, Section 7 of the United States Constitution The Presentment Clause of Article I, Section 7 of the United States Constitution The Prohibition on bills of attainder in Article I, Section 9 of the United States Constitution The Takings Clause of the Fifth Amendment to the United States Constitution

Answer choice B is correct. The Presentment Clause requires any bill that has been passed by Congress to be presented to the President. If the President signs the bill, it becomes law. If the President does not, it may either become law or not depending on whether the President vetoes it (either directly or via a "pocket veto"), and if by a direct veto, whether Congress overrides the President's veto. The federal law that effectively gives the President a so-called "line-item veto" thwarts the procedure set out in the Presentment Clause. Answer choice A is incorrect because the Origination Clause of Article I, Section 7 requires that a bill for raising revenue originate in the House, rather than the Senate. The law being challenged, the "line-item" veto statute, was not a revenue bill. Therefore, the Origination Clause is unlikely to apply. Answer choice C is incorrect. Although Article I, Section 9 does prohibit Congress from enacting a bill of attainder (i.e., a legislative act that declares a person or group of persons guilty of some crime and punishes them without a trial), the "line-item" veto statute does not impose a criminal penalty on the recipient of a tax benefit. Instead, the statute merely allows the President to deny the recipient that tax benefit by vetoing only that benefit. Answer choice D is incorrect because the Takings Clause of the Fifth Amendment applies to the taking of private property for a public purpose without just compensation. The recipient of the tax credit does not have a property interest in that tax credit, particularly a tax credit that is void before it becomes law.

Question 8584 A wastewater treatment facility was fined for violation of a state law regarding the discharge of pollutants into water sources. The facility challenged the penalty in state court on the basis that the state law conflicted with federal law. In the court's opinion, which sided with the facility, the court analyzed the state statute in light of federal law regarding a variety of issues related to pollution and the water source into which the pollutants were discharged, but failed to clearly indicate the basis for its decision. On appeal, the decision was affirmed per curiam by the highest court of the state. The state appealed the decision to the U.S. Supreme Court. If the Supreme Court were to review this case, what would be the most likely reason allowing such review? Answers: Federal law would consider the discharge from the facility to be illegal pollution. The state court decision incorrectly applied state law. The state court decision relied on federal law. The U.S. Supreme Court is required to hear the case.

Answer choice C is correct. A final state court judgment that rests on adequate and independent state grounds may not be reviewed by the U.S. Supreme Court. The state law grounds must fully resolve the matter (i.e., be adequate) and must not incorporate a federal standard by reference (i.e., be independent). When it is not clear whether the state court's decision rests on state or federal law, the Supreme Court may hear the case, decide the federal issue, and remand to the state court for resolution of any question of state law. Answer choice A is incorrect because, regardless of whether the facility's discharge of the pollutants was illegal, the Supreme Court's ability to review a final state judgment would hinge on whether the decision was based on adequate and independent state grounds. Here, the decision cited both federal and state law, allowing for the Supreme Court to review the decision. Answer choice B is incorrect because the Supreme Court generally lacks jurisdiction to review a state court decision applying state law. Answer choice D is incorrect because the Supreme Court may hear the case but is not required to do so.

Question 3043 During a defendant's civil trial for assault, a witness for the plaintiff testified that, shortly after the assault took place, the defendant had admitted to her that he did assault the plaintiff. On cross-examination, the defendant's counsel asked the witness whether she had testified in the defendant's prior criminal prosecution for assault of the plaintiff in the current case that the defendant told the witness that he had never even met the plaintiff. The plaintiff objected to the defendant's question. May the court allow the question over the plaintiff's objection? Answers: Yes, but only to prove that the defendant did not commit the assault. Yes, but only to impeach the credibility of the witness. Yes, both to prove that the defendant did not commit the assault and to impeach the credibility of the witness. No, because the defendant did not first allow the witness the opportunity to explain or deny the statement.

Answer choice C is correct. A witness's prior inconsistent statement is admissible to impeach the witness's credibility. As substantive evidence introduced for its truth, the witness's prior statement involves two statements—what the witness testified to at the defendant's prior criminal trial and what the defendant told her. The witness's prior testimony is admissible substantively as non-hearsay because her statement was made under oath at the prior criminal proceeding. The defendant's statement that he had never met the plaintiff is also admissible as non-hearsay because it is a statement by a party-opponent. Answer choices A and B are incorrect because each only recognizes one of the two admissible uses for the evidence. Answer choice D is incorrect because, while the witness must be given the chance to explain or deny the inconsistent statement, the opportunity to explain or deny need not take place before the statement is admitted into evidence.

Question 5032 A woman was killed when a business's delivery truck swerved onto the sidewalk at a high rate of speed. The woman's husband brought an action for wrongful death under state law against the business owner in federal district court based on diversity jurisdiction. On October 1, the jury returned a verdict in favor of the business owner and judgment was entered. After trial, the court learned that one of the jurors had discussed the case with the business owner and the business owner's attorney outside of court during the trial. The juror had become friendly with the business owner and decided that he would influence the jury deliberations and convince the jury to find in favor of the business owner. On October 15, after considering what it had learned, the court, on its own initiative, ordered a new trial. The order did not contain any information regarding the juror's actions. Were the court's actions proper? Answers: Yes, because the juror's actions constituted prejudicial misconduct. Yes, because the order was made within 28 days after the entry of judgment. No, because the order did not specify the reasons for the new trial. No, because the court can only order a new trial upon a motion by one of the parties

Answer choice C is correct. The court, on its own initiative, may order a new trial for any reason that would justify granting a new trial on the motion of a party. The court, though, must specify the reasons in its order. Here, the court was permitted to order a new trial on its own initiative due to the prejudicial misconduct of the juror. However, the court was required to state the grounds for the new trial in its order. Because the court failed to do so, the court's actions were not proper. Answer choice A is incorrect. While the juror's actions constituted prejudicial misconduct and would be a valid ground for ordering a new trial, the court's failure to specify the reasons for granting a new trial in its order make its actions improper. Answer choice B is incorrect because the fact that the court's order was timely does not automatically make it valid. Answer choice D is incorrect because the court may order a new trial on its own initiative, without a motion from one of the parties.

Question 6243 A teenage girl wanted to be popular in high school. A group of "mean girls" approached her and asked her to join their clique. The teenage girl quickly agreed. That weekend, the mean girls held a party at which they told the teenage girl that she had to kill their math teacher to be in the group. The teenage girl refused and said she did not want to be in the clique after all. The mean girl leader then said to the other members, "She's already in, right? We can't let her out. We'll kill her if she tries to leave." The other members all orally agreed. On Monday morning, the teenage girl put poison in the math teacher's coffee cup, and he died while explaining quadratic equations. The teenage girl is charged with deliberate and premeditated murder. If the teenage girl establishes that she killed the math teacher because the mean girls threatened to kill her, should she be convicted? Answers: No, because she was forced to act under duress. No, because the killing of the math teacher was a legal necessity. Yes, because duress is never a defense to intentional murder. Yes, because the mean girls threatened the teenage girl with future, not immediate harm.

Answer choice C is correct. The teenage girl is guilty of deliberate and premeditated murder because she killed the math teacher intentionally and with premeditation. She cannot claim duress because duress is never a defense to intentional murder. Answer choice A is incorrect. Even though the teenage girl was coerced into committing the crime, duress is never a defense to intentional murder. Answer choice B is incorrect because the defense of necessity only applies when natural, non-human forces (e.g., weather) require the defendant to commit the criminal act. Answer choice D is incorrect because it refers to the requirements of self-defense. Self-defense does not excuse the use of deadly force against a third party in lieu of an actual aggressor. Therefore, even a threat of immediate harm by the mean girls would not defend the teenage girl's violence against the teacher.

Question 7355 The guest of a member of a condominium association was injured when he slipped on a wet spot in the front lobby area. This area is owned by each unit owner as a tenant-in-common with the other unit owners. The guest sued the association, and a jury determined that the association was liable for injury to the member's guest. The jury awarded the guest damages. Which of the following best describes the responsibility of each member for the damages? Answers: No member bears any financial responsibility for the damages. Each member is jointly liable for the total amount of the damages. Each member is jointly and severally liable for the total amount of the damages. Each member is liable for her share of the association expenses, which includes payment of the judgment awarded to a tort plaintiff.

Answer choice D is correct. Although each member of a condominium association owns the common areas of the condominium as a tenant-in-common with the other members, the member is only indirectly responsible for a tort judgment against the association. Payment of the tort judgment by the association is an expense of the association, and each member is liable for a share of the association's expenses. For this reason, answer choices B and C are incorrect. Answer choice A is incorrect because while a member is not directly responsible for payment of a tort judgment awarded against the association, the member does bear a portion of the financial responsibility for the judgment as an association expense. Consequently, answer choice D is the better alternative.

Question 8432 The owner of a store that has operated at a loss since it was opened several years ago needs additional capital to keep the store open. She approached a friend about investing in the store. The friend, who is an unsophisticated investor, asked about the store's profitability. The owner told her that it is a "money maker." The friend, relying on the owner's statement and without further investigation, agreed to invest in the store in exchange for an ownership interest in the store. Six months later, the friend, learning that the store is unprofitable, sued the owner to rescind their agreement on the grounds of fraudulent misrepresentation. Will the friend be successful? Answers: No, because the friend failed to further investigate the store's profitability. No, because the owner's statement that the store was a "money maker" was only her opinion as to the value of the business. Yes, because the owner exercised undue influence over the friend. Yes, because the owner knew that the store was not profitable.

Answer choice D is correct. Fraudulent misrepresentation requires proof of the following: (i) the misrepresentation is fraudulent, meaning (a) a false assertion of fact made knowingly, or recklessly without knowledge of its truth, and (b) with intent to mislead the other party; (ii) the misrepresentation induced assent to the contract; and (iii) the adversely affected party justifiably relied on the misrepresentation. Generally, an assertion of an opinion, such as a belief or judgment as to the quality, value, or authenticity of an item or the occurrence of a future event, is not an assertion of a fact. However, an assertion of an opinion may, if reasonable, be interpreted by a party as an assertion that the person knows facts that are not incompatible with the opinion or that the person knows facts sufficient to justify the formation of the opinion. Here, the store owner's assertion that the store was a "money maker" was an opinion about the store's value, but it was also an assertion that the owner knew facts sufficient to justify that opinion. However, the store owner knew that the store had never been profitable. The statement was thus a false assertion of the store's profitability that was made knowingly with the intent to mislead the friend, the statement induced the friend to invest in the store, and the friend justifiably relied on the statement. Answer choice A is incorrect. A party's fault in not knowing or discovering facts before entering into a contract does not prevent the party's reliance on the misrepresentation from being justified unless it constitutes a failure to act in good faith and in accordance with the reasonable standards of fair dealing. A party's negligence with regard to learning about the falsity of the misrepresentation is not sufficient to prevent the party from avoiding the contract. Here, the friend's failure to further investigate the store's profitability does not prevent the friend from avoiding the contract. Answer choice B is incorrect. The store owner's assertion that the store was a "money maker" was an opinion about the store's value, but it was also an assertion that the owner knew facts sufficient to justify that opinion. Answer choice C is incorrect. Undue influence is an independent ground upon which a contract can be avoided, not a requirement for fraudulent misrepresentation, which is the ground upon which the friend has sued the owner.

Question 7448 A computer programmer for a local software company was upset by the loud parties held in the apartment next door by a college student whenever her father, with whom she was living for the summer, went out of town. One day, while no one was home, the programmer broke into the apartment and installed a video camera. Operating the camera from his apartment, the programmer recorded the college student's next party. The recording contained evidence of the sale and use of illegal drugs by the college student. The programmer provided that recording to the police, who arrested the college student for the distribution of illegal drugs. The college student has moved to suppress the recording as a violation of her Fourth Amendment rights. How should the court rule? Answers: Grant the motion, because the programmer illegally gained access to the college student's apartment. Grant the motion, because the college student had a reasonable expectation of privacy in the apartment. Deny the motion, because the recording establishes that the college student was distributing illegal drugs. Deny the motion, because the police had no role in making the recording.

Answer choice D is correct. Since the recording was not made by or at the behest of the police but by a private individual, the Fourth Amendment does not prohibit the government's use of the recording at the college student's trial. Answer choice A is incorrect because, while the college student has a private cause of action against the programmer for trespass and invasion of privacy, the illegality of the programmer's acts to not prevent the use of the recording at trial. Because the recording is not the product of police conduct or conduct engaged in by a private citizen under the direction of the police, the Fourth Amendment does not require its suppression. Answer choice B is incorrect because, while even a temporary resident of a dwelling has a reasonable expectation of privacy in the dwelling, here the recording was produced by the programmer, a private citizen, and not by the police. Answer choice C is incorrect because the relevance of the recording to charges brought against the college student would permit the recording, if properly authenticated, to be introduced at trial; however, relevancy is not at issue in this motion. Instead, the issue is whether the recording was produced in violation of the college student's Fourth Amendment rights, which it was not.

Question 7525 A defendant was charged with assault after being involved in a barroom fight in the middle of the day. The defendant admitted to being at the bar at the time of the fight, but claimed that he was only a bystander. At the defendant's trial, the prosecutor intended to call the defendant's former employer. The employer was to testify that the defendant had been fired and was not working at the time of his arrest. The defendant objected to the employer's testimony. How should the court rule? Answers: Overrule the objection, because it tends to make it more likely that the defendant was at the bar in the middle of the day and involved in the fight. Overrule the objection, because the employer will be testifying based on his personal knowledge of the defendant's employment. Sustain the objection on the basis that the employer's testimony is unfairly prejudicial. Sustain the objection on the basis that the employer's testimony is not probative of a material fact.

Answer choice is C is correct. As a general rule, evidence must be relevant in order to be admissible. Evidence is relevant if (i) it has any tendency to make a fact more or less probable than it would be without the evidence (i.e., probative) and (ii) the fact is of consequence in determining the action (i.e., material). In this case, the defendant is on trial for assault. The fact that the defendant was unemployed at the time of his arrest makes it more likely that he was at the bar (and therefore potentially involved in the fight). However, under FRE 403, relevant evidence may be excluded if its probative value is substantially outweighed by the danger of unfair prejudice. Here, the employer's testimony includes the fact that the employee was fired. This fact is highly prejudicial, as it would tend to discredit the defendant in the minds of the jury. Moreover, the probative value of the employer's testimony is somewhat minimal. Accordingly, even though it is relevant, the employer's testimony should not be admitted. Answer choice A is incorrect because, as stated above, the fact that the defendant is unemployed does make it more likely that he was at the bar and potentially involved in the fight. However, although relevant, the employer's testimony is likely unfairly prejudicial. Answer choice B is incorrect. Although the employer is testifying based on his own personal knowledge as to a matter within the scope of lay witness testimony, the testimony is unfairly prejudicial. Answer choice D is incorrect because the employer's testimony regarding the defendant's unemployment is relevant to whether the defendant was present at the bar (and therefore potentially involved in the fight). The mere fact that the defendant has stipulated to his presence at the bar does not automatically preclude the prosecution from presenting the employer's testimony, which also suggests the defendant was at the bar. However, such testimony should be excluded because it is unfairly prejudicial.

Question 5045 The defendant lives in State A. He commutes to work in nearby State B, where he does most of his shopping (State B has no sales tax). The defendant spends a month each summer vacationing in State C. While driving from home to State C, Defendant traveled through State D for the first time, where he negligently hit and totaled the $100,000 sports car of the plaintiff, a State D citizen. The defendant continued on to State C, where he vacationed for several weeks. The day after the accident, the plaintiff filed a complaint in federal court and sought to serve process on the defendant the next day, either personally or by mail. States A, B, C, and D all have statutes authorizing personal jurisdiction over anyone who engages in activity within the state, to the extent permissible under the U.S. Constitution. Assuming subject-matter jurisdiction is proper, in which state is the plaintiff least likely to find personal jurisdiction over the defendant? Answers: State A. State B. State C. State D.

Answer choice B is correct. Defendant is neither domiciled nor present in State B, so on the day after the accident, State B would have no personal jurisdiction over him. Answer A is incorrect because personal jurisdiction exists in the state in which the defendant is domiciled, which is likely State A, the state in which he lives. Answer C is incorrect because the defendant is voluntarily present in State C and may be served with process there. Answer D is incorrect because, under the long-arm statute of State D, the defendant may be sued in State D. By driving his car there, D may be sued for an accident that resulted from that activity.

Question 7484 A security guard, who was a retired police officer, witnessed a traffic accident. He left his name with one of the parties to the accident. A week later, the attorney for that party contacted him. When the guard met with the attorney about the accident, the guard, as was his habit from his days as a police officer, drew a map of the place where the accident happened that contained various details about the accident. The guard labeled the map with the time and date of the accident and then signed and dated it. The attorney filed a negligence action on behalf of her client against the other party to the accident. Prior to the trial, the security guard died. The attorney, unable to locate the original map drawn by the security guard, seeks to introduce a properly authenticated photocopy of the map into evidence as proof of how the accident occurred. Is this photocopy of the map admissible? Answers: No, because it is inadmissible hearsay. No, because it violates the original document rule. Yes, as habit evidence. Yes, as past recollection recorded.

Answer choice A is correct. Hearsay is an out-of-court assertion that is offered to prove the truth of the matter asserted. In addition to written and oral statements made by the declarant, nonverbal conduct by the declarant, such as a map drawn by a witness to the accident that depicts the scene of the accident, is an assertion. Consequently, the map and its photocopy are inadmissible hearsay. Answer choice B is incorrect. The original document rule requires that the original document (or a reliable duplicate) be produced in order to prove the contents of a writing, recording, or photograph. A duplicate is a counterpart produced by any process or technique that accurately reproduces the original, and is generally admissible to the same extent as an original unless there is a genuine question as to the authenticity of the original. Since the photocopy is a duplicate of the original map and it has been properly authenticated, the photocopy does not violate the original document rule. Answer choice C is incorrect because, while habit evidence is admissible to prove action in conformity with that habit, here, the security guard is not a party to this action and his habit of drawing a map of scene of accident is not relevant to the negligence of the parties to the accident. Answer choice D is incorrect because, while past recollection recorded is an exception to the hearsay rule, it requires, among other things, that the witness testify that the record accurately reflects the witness's knowledge. Since the security guard is dead, he cannot testify to this fact.

Question 8368 In a state court of general jurisdiction in State A, a debtor, who was a resident of State A, brought suit against a lender, who was an alien with permanent residency status in State B, for violation of the federal Fair Debt Collection Practices Act. The lender waived personal jurisdiction. Under state procedural rules, venue was proper in the state court. The lender timely removed the action to the federal court for the Northern District of State A, which was the district in which the state court sat. The lender then timely moved to dismiss the action for improper venue, noting that, had the action initially been brought in federal court, the federal court for the Northern District of State A would not have had proper venue over this action. How should the court rule on this motion? Answers: Deny the motion, because the state court sat in the federal Northern District of State A. Deny the motion, because venue was proper in the state court under state procedural rules before the action was removed. Grant the motion, because the lender is deemed to reside in the judicial district where that person is domiciled. Grant the motion, because the federal court for the Northern District of State A would not have had proper venue over this action had the action been initially brought in federal court.

Answer choice A is correct. In a case that is removed from state court, venue is proper in the federal district court in the district where the state action was pending. Since this action was removed from a state court in the Northern District of State A, venue is proper in the federal district court for the Northern District of State A. Answer choice B is incorrect because the federal rules regarding proper venue when a case is removed from state court do not turn on whether venue was proper in the state court from which the case was removed. Answer choice C is incorrect because, while an alien lawfully admitted for permanent residence into the United States is deemed to reside in the judicial district where that person is domiciled, which in this case is State B, in a case that is removed from state court, venue is proper in the federal district court in the district where the state action was pending. Answer choice D is incorrect because, in a case that is removed from state court, venue is proper in the federal district court in the district where the state action was pending. It is immaterial that venue would not have been proper under the general venue rule if the action had been brought initially in that district.

Question 7515 An elderly widow conveyed her fee simple interest in a vacation home as a wedding present to her son and his wife as joint tenants with the right of survivorship, which the couple gladly accepted but did not record the deed. This marriage was the second marriage for each. They had no children together, but each had one child from a prior marriage. The husband, the widow's son, had a son; the wife, the widow's daughter-in-law, had a daughter. On their honeymoon, the husband and wife were killed in a boating accident. There was clear and convincing evidence that the husband survived his wife by several hours. The wife had a will in which all her real property was devised to her daughter. The husband did not have a will; his son was his only heir. The applicable jurisdiction has the following statute: If it is not established by clear and convincing evidence that one of two co-owners with the right of survivorship survived the other co-owner, one-half of the co-owned property passes as if one had survived and one-half as if the other had survived. Who now owns the vacation home? Answers: The husband's son, as the husband's heir. The wife's daughter, under the terms of the wife's will. The husband's son and the wife's daughter each own a one-half interest in the vacation home as tenants in common. The widow, because the deed was not recorded.

Answer choice A is correct. In a joint tenancy with the right of survivorship in which there are only two co-tenants, when one of the joint tenants dies before the other, the property held as joint tenants becomes the property of the surviving joint tenant unless an applicable statute mandates a different result. Here, there is a statute that modifies this rule when there is not clear and convincing evidence of one co-tenant surviving the other co-tenant. However, as there is such evidence in this case, the vacation home passes by intestacy to the husband's only heir, his son. Answer choice B is incorrect. Although a person who leaves a will that devises property to a beneficiary can override the disposition of that property to the person's heirs under the intestacy laws, a person's will has no direct effect over the disposition of property that is owned by someone else as a consequence of the person's death, even if the other person did not have a will. Here, the vacation home became solely the husband's property upon the death of his wife, and passed, as he did not have a will, to his son, as the husband's only heir in intestacy. Answer choice C is incorrect. Had there not been clear and convincing evidence that either the husband or his wife survived the other, the quoted statute would have dictated this result, in effect converting the jointly held property into a tenancy in common. However, because there was clear and convincing evidence that the husband survived his wife, the statute is not applicable here. Answer choice D is incorrect because a deed need not be recorded to be valid and convey good title. Here, even though the deed was not recorded, the deed was delivered to the husband and wife and they accepted it, thereby completing the gift transfer.

Question 374 The defendant was convicted of first-degree murder. He sought out and stabbed his victim multiple times after a dispute over a soccer match. The prosecution sought the death penalty. The statute in effect states that the death penalty is reserved for only those crimes that are "especially heinous or brutal." The defendant was sentenced to death. He is now appealing his sentence. Should the appellate court vacate his sentence? Answers: Yes, because the requirement that death penalty crimes be "especially heinous or brutal" is unconstitutionally vague. Yes, because the only permissible statutory criteria for determining whether the death penalty should be applied is whether the murder was premeditated. No, because the "especially heinous or brutal" requirement does not describe a characteristic that applies to every murder defendant. No, because a reasonable jury could determine that the murder was in fact especially heinous or brutal.

Answer choice A is correct. In order to impose the death penalty, the aggravating circumstance involved cannot be unconstitutionally vague. A crime being "heinous or brutal" has been held unconstitutionally vague. Answer choice B is incorrect because, although many jurisdictions use this as the criteria for the death penalty, it is not the only permissible criteria. Answer choice C is incorrect because the standard is vague. It does not matter that it passes the first prong of the aggravated circumstances test, whether the circumstance sets the murder apart from others. Answer choice D is incorrect because the fact that a reasonable jury could conclude that the murder was heinous is not enough to render the requirement constitutional.

Question 403 A defendant was found guilty of murder. With regard to the separate sentencing phase of the trial to determine whether the death penalty should be imposed, the defendant's attorney did not take the following actions: obtain a social history of the defendant at state expense, which defense attorneys in the state routinely obtained; present witnesses who had testified on behalf of the defendant in the guilt phase due to their ineffective performance during the guilt phase; present mitigating evidence which was subject to an adverse interpretation by the jury; and make a closing argument in the sentencing phase in order to prevent a rebuttal argument by a persuasive prosecutor. As described above, which of these actions is most likely to give rise to an ineffective assistance of counsel claim? Answers: Failure to obtain defendant's social history. Failure to present witness. Failure to present mitigating evidence. Failure to make a closing argument.

Answer choice A is correct. The first part of the two part test for establishing ineffective assistance of counsel under the Due Process Clause of the Fourteenth Amendment requires that the representation of a defendant by the defendant's attorney fall below an objective standard of reasonableness. There is a presumption of competent representation. An attorney who has failed to utilize an investigative device used routinely by other defense attorneys in the state may be considered not to have met the reasonableness standard. Answer choices B, C, and D each describe a tactical choice made by the defense attorney. Tactical choices rarely fail to meet the standard of reasonableness so long as the attorney does have a tactical reason for making such a choice. Each of these choices reveal such a reason.

Question 5892 A homeowner properly executed a valid will devising his primary residence to his girlfriend and the residue of his estate to his life-long friend. The life-long friend is the executor of the homeowner's estate and knows that the homeowner intended to provide a home for his girlfriend. The value of the homeowner's primary residence increased substantially following the execution of his will. After receiving a favorable offer, the homeowner decided to sell his primary residence and move in with his girlfriend. Years later, the homeowner died. The girlfriend claims a right to the proceeds from the sale of the homeowner's primary residence under the terms of the will. Is she likely to succeed? Answers: No, because the devise to the girlfriend lapsed. No, because the devise to the girlfriend adeemed. Yes, because the proceeds from the sale are traceable. Yes, because there is extrinsic evidence of the homeowner's intent.

Answer choice B is correct. A devise of real property may fail because the testator no longer owns the property upon death. If the testator sells, destroys, or gives away property before his death, a devise of that property is adeemed and the intended beneficiary takes nothing. In this case, the homeowner attempted to make a specific devise of his primary residence to his girlfriend. However, before his death, the homeowner sold the property. Therefore, the property has adeemed and the girlfriend takes nothing under the homeowner's will. Answer choice A is incorrect because the gift has not lapsed. Lapse occurs when the intended beneficiary predeceases the testator. Here, the girlfriend is still alive at the time of the homeowner's death, so the devise has not lapsed. Answer choice C is incorrect because it is irrelevant whether the proceeds are traceable. A specific devise transfers a distinguishable piece of property and will not be substituted if the property has adeemed. In this case, the homeowner no longer owns the primary residence and cannot devise property he does not own. Answer choice D is incorrect because there are no ambiguities in the will. Extrinsic evidence of the testator's intent will not be allowed when the testator's intent is clear from the will.

Question 6693 A defendant has been charged with battery. While waiting to bat in a game between two amateur teams in a recreational softball league, the defendant was taking practice swings. Unlike the other players in the league, who took care to look behind them before taking a practice swing, the defendant undertook no similar precaution. After stopping for about a minute, the defendant resumed his practice swings. On the follow through of his first practice swing after restarting, the defendant carelessly struck a team member in the head, causing serious injury. The team member, who was facing away from the defendant at the time, had inadvertently moved into the path of the defendant's bat, unaware that the defendant had resumed his practice swings. Is the defendant likely guilty of battery? Answers: Yes, even though the victim had impliedly consented to the harmful contact by participating in the softball game. Yes, because the defendant's failure to take precautions was a gross deviation from the standard of care of a reasonable person in the same situation. No, because the victim was unaware that the defendant had resumed his practice swings and that he was about to be hit. No, because the defendant lacked the specific intent to harm the victim.

Answer choice B is correct. Battery is the unlawful application of force to another person that causes bodily harm or constitutes an offensive touching. Battery is a general-intent crime that includes not only intentional conduct but also criminal negligence. Here, the defendant's practice swing caused a serious head injury to a team member. His failure to take a precaution undertaken by the other members in the softball league suggests that this failure was not mere negligence, but instead was a gross deviation from the standard of care of a reasonable person in the same situation, thus constituting criminal negligence. Answer choice A is incorrect. Although the team member is deemed to have consented to harmful contact that results from participating in the softball game, such as being tagged out by an opposing player or being hit by a softball on the field, it is unlikely that such consent extends to conduct that does not occur as a direct consequence of playing the game itself. Answer choice C is incorrect because battery does not require that the victim be apprehensive of an imminent bodily harm or offensive contact. Answer choice D is incorrect. Although a defendant who acts with the specific intent to cause bodily harm or an offensive touching can be guilty of battery, specific intent is not required. Battery is a general-intent crime that includes not only intentional conduct but also criminal negligence.

Question 385 A defendant is charged as a principal with the rape of a 10-year-old child and with murder, under a felony murder statute, for her unintended death, which occurred during the commission of the rape. Pursuant to state law, the prosecutor timely notifies the defendant of the state's intent to seek the death penalty. The defendant challenges the prosecutor's decision on the grounds that the imposition of the death penalty for these crimes constitutes cruel and unusual punishment under the Eighth Amendment. Should the court permit the prosecutor to seek the death penalty for these crimes? Answers: Yes, as to both rape and felony murder. No, as to rape, but yes as to felony murder. No, as to felony murder, but yes as to rape. No, as to both rape and felony murder.

Answer choice B is correct. Capital punishment for the crime of rape, even when the victim is a child, constitutes cruel and unusual punishment under the Eighth Amendment. Accordingly, answer choices A and C are incorrect because they both would allow capital punishment for the crime of rape. Answer choice D is incorrect because significant participation in the commission of the felony that resulted in the murder, plus a reckless indifference to human life, is sufficient to satisfy the culpability requirement to justify the death penalty--even absent the intent to kill. Here, although the defendant did not intend the rape victim's death, the defendant committed the violent felony that resulted in her death.

Question 6287 A state statute imposes a special tax on the receipts of publishers from the sale of magazines within the state. The statute exempts religious, professional, and trade magazines from this tax. The publisher of a general-interest magazine filed suit in federal court contending that the selective application of this tax was unconstitutional. Is the court likely to hold in favor of the plaintiff? Answers: Yes, because publishers enjoy greater First Amendment rights than the general public. Yes, because the selective application of the tax violates the First Amendment rights of the plaintiff. No, because unlike the federal government, a state is not confined to the exercise of specifically enumerated powers. No, because the selective application of the tax is rationally related to a legitimate purpose.

Answer choice B is correct. Due to the exemption granted to religious, professional, and trade magazines, the selective application of the tax constitutes a content-based restriction on the plaintiff's speech. As such, it is subject to strict scrutiny. Consequently, the state is unlikely to be able to establish that it has a compelling governmental interest that the selective tax system is necessary to achieve, and that this system is narrowly tailored to meet that interest. Answer choice A is incorrect because publishers do not enjoy greater First Amendment rights than the general public. A publisher is subject to a generally applied tax to the same extent as any other citizen would be. Answer choice C is incorrect because, while it is true that a state, unlike the federal government, is not confined to the exercise of specifically enumerated power, a state is subject to the constraints of the First Amendment via the Fourteenth Amendment. Therefore, this rule is not determinative here. Answer choice D is incorrect because, even assuming that the selective application of the tax is rationally related to a legitimate purpose, this is the incorrect standard to apply in assessing the constitutionality of this state action. Instead, because the selective application of the tax is a content-based restriction on free speech, it must instead satisfy the strict scrutiny test.

Question 7319 On a warm summer afternoon, a man was sitting on a park bench looking at his cell phone. Beside him, he had placed a box with a pair of handmade shoes for which he had paid $1,000 earlier that day. A jogger came by and picked up the box before the man was aware of what was happening. Immediately afterwards, the man gave chase. As the jogger turned and fired a gun at the man, the man stumbled. The bullet struck and killed a woman who was strolling behind the man in the park. The jogger was arrested and charged with felony murder of the woman. In the applicable jurisdiction, larceny of personal property with a value of more than $500 is a felony. Can the jogger be convicted of felony murder of the woman? Answers: No, because the jogger did not intend to kill the woman. No, because the jogger did not commit an inherently dangerous felony. Yes, under the doctrine of transferred intent. Yes, because the killing occurred during the jogger's flight from the scene of the crime.

Answer choice B is correct. Felony murder is an unintended killing proximately caused by and during the commission or attempted commission of an inherently dangerous felony. The felonies traditionally considered inherently dangerous are: burglary, arson, rape, robbery, and kidnapping. Here, although the jogger clearly committed larceny (i.e., the trespassory taking and carrying away of the personal property of another with the intent to permanently deprive that person of the property) and this crime was a felony as the value of the shoes exceeds $500, the jogger did not commit a robbery, which requires that the defendant commit the larceny through the use of force or intimidation. The jogger gained possession of the shoes without the use of force or intimidation. Consequently, the jogger did not commit an inherently dangerous felony and therefore cannot be convicted of felony murder. Answer choice A is incorrect. A defendant may be convicted of felony murder, even though the killing is unintentional. Accordingly, the jogger's lack of intent to kill the woman would not prevent the jogger from being convicted of felony murder. Answer choice C is incorrect. Although the doctrine of transferred intent does apply to murder, and would be applicable under these facts if the jogger were charged with murder, the jogger was charged with felony murder and felony murder does not require that the killing be committed with intent. Instead, the fact that the killing occurs during the commission of an inherently dangerous felony supplies the necessary mens rea to permit the defendant to be convicted of felony murder. Answer choice D is incorrect. Although a killing that occurs during a defendant's flight from the scene of a crime is considered to have occurred during the commission of the crime for purposes of felony murder, the jogger's crime here, felony larceny, is not an inherently dangerous felony.

Question 5976 A plaintiff was injured when he fell in the parking lot of a chemical plant. The chemical plant, located in an uninhabited area, stored dangerous toxic chemicals that needed to be specially contained. The chemical plant exercised the utmost care in maintaining and containing the chemicals. At the time of his injury, the plaintiff was walking towards the entrance of the chemical plant. Distracted by his cell phone ringing, the plaintiff stepped into a pothole and twisted his knee. The plaintiff brought a strict liability action against the chemical plant, seeking damages for his injury. The plaintiff can establish that the plant failed to exercise reasonable care in maintaining the parking lot. Can the plaintiff recover? Answers: No, because the chemical plant exercised the utmost care in conducting its storage activities. No, because the plaintiff's injury did not result from an abnormally dangerous activity. Yes, because the chemical plant didn't exercise reasonable care in maintaining the parking lot. Yes, because the chemical plant engaged in an abnormally dangerous activity.

Answer choice B is correct. Strict liability for an abnormally dangerous activity exists only if the harm that occurs results from the risk that made the activity abnormally dangerous. In this case, stepping into a pothole caused the plaintiff's injury. The injury did not result from toxic chemicals escaping from the storage facility (i.e., the risk that made the operation of the chemical plant abnormally dangerous). Therefore, the plaintiff cannot recover in a strict liability action against the chemical plant for the injury to his knee. Answer choice A is incorrect. The degree of care utilized by the chemical plant to prevent harm is not relevant in an action for strict liability based upon an abnormally dangerous activity. Answer choice C is incorrect. The chemical plant's failure to maintain the parking lot in a safe condition would be relevant to a claim for negligence based upon the chemical's plant failure to maintain the parking lot. However, it is not relevant in an action based upon strict liability. Answer choice D is incorrect. Although the storage of toxic chemicals is considered an abnormally dangerous activity, the carrying on of that activity at the plant does not automatically lead to liability for any injury that occurs on the premises of the plant. The injury in question must arise from the risk that made the chemical plant's activity abnormally dangerous.

Question 8440 A couple leased a residence for one year beginning on January 1. The rent for the term was $30,000, payable in advance in monthly increments of $2,500 on the first of each month. On August 1, the couple notified the landlord that they were leaving at the end of the month. They moved out on August 31, leaving the residence vacant. When the couple left, they had not paid the rent for the months of July and August. The landlord took no action regarding the residence, but instead simply allowed it to remain vacant. Had the landlord made a reasonable effort to rent the residence, the residence could have been rented for $2,000 per month for the remaining term of the couple's lease. The jurisdiction has adopted a mitigation rule regarding abandoned leased premises. In January of the following year, the landlord sued the couple for damages under the lease. How much should the landlord receive in damages? Answers: $15,000 $7,000 $5,000 $2,000

Answer choice B is correct. The couple's lease created a tenancy for years with a term of one year. A tenancy for years terminates automatically at the end of the term. A party is not required to give the other party notice of the termination of the tenancy, and notice by the tenants that they are leaving the premises early does not terminate the tenancy. However, in jurisdictions that recognize the mitigation rule regarding a tenant's abandonment of the leased premises, a landlord, upon learning of a tenant's abandonment of the premises, has a duty to mitigate damages by making a reasonable attempt to rent the premises, and failure to do so will reduce the landlord's recoverable damages by the amount the landlord could have mitigated the damages. Here, had the landlord made such an attempt, he would have received $8,000 ($2,000 per month for four months) instead of the $10,000 that he would have received from the couple ($2,500 per month for four months). Consequently, the landlord is entitled to only $2,000 in damages for the final four months of the lease term. When that amount is added to the unpaid rent of $5,000 for the months of July and August ($2,500 per month for two months), the landlord is entitled to damages of $7,000. Answer choice A is incorrect because this amount fails to consider the landlord's duty to mitigate damages by renting the residence at $2,000 per month for the last four months of the year. Answer choice C is incorrect because it fails to recognize the damages to which the landlord is entitled for the last four months of the lease term. Answer choice D is incorrect because it fails to include the rent owed for July and August.

Question 369 A defendant in a robbery case was awaiting trial, though his attorney had negotiated with the prosecution to allow him to be under house arrest while awaiting trial, instead of sitting in jail. The police had been unable to find the gun that the defendant purportedly used to commit the robbery, and knew that finding the gun would greatly help the prosecution make its case. One of the police officers decided to visit the defendant at his home. After talking with the officer for about 30 minutes, the defendant decided to confess to the robbery. The defendant also admitted that the gun was hidden in a random office building downtown, which the police had never thought to search. Based on the confession, the officer got a warrant to search the building and found the gun. The prosecutor plans to introduce both the confession and the gun into evidence at the defendant's trial. On a motion by the defense to suppress the confession and the gun, how should the court rule? Answers: Grant the motion as to the confession, but deny it as to the gun. Grant the motion as to both the confession and the gun. Deny the motion as to the confession, but grant it as to the gun. Deny the motion as to both the confession and the gun.

Answer choice B is correct. The evidence should be suppressed because there were both Fifth and Sixth Amendment violations. The defendant's Sixth Amendment rights were violated because he was represented by counsel in the matter at issue and there was no effective waiver of that right. Evidence obtained in violation of a defendant's Sixth Amendment rights may not be introduced at trial. This is the case for both statements of a defendant and any evidence derived from such statements. In addition, the defendant's Fifth Amendment rights were violated. Since the defendant here was under house arrest, he was in "custody." A reasonable person would not believe that he was free to leave under such circumstances. Since there were no Miranda warnings given, the confession is inadmissible. (Note that the lack of Miranda warnings would not necessarily preclude admission of the gun.) Answer choice A is incorrect because it would allow for admission of evidence derived from such a confession. Answer choices C and D are incorrect because they would allow for admission of an illegally obtained confession.

Question 7248 A defendant was charged with criminal battery in federal court for allegedly breaking his wife's arm while they lived together before they were married. At the time of the alleged battery, the wife reported the incident to the police and moved to her sister's house while her arm healed. She and the defendant then reconciled and were married. The prosecution has called the wife to testify at the trial. Which of the following most accurately states the rule as to whether the wife may testify? Answers: The wife can be compelled to testify unless the defendant objects. The wife can be compelled to testify, even if the defendant objects. The wife can choose to testify as long as the defendant does not object. The wife can choose to testify, but cannot be compelled to do so.

Answer choice B is correct. The general rule is that the spouse of a criminal defendant may not be called as a witness by the prosecution. A married person may not be compelled to testify against her spouse in any criminal proceeding, regardless of who is the defendant. However, there is an exception when one spouse is charged with a crime against the other spouse or the children of either. Therefore, the wife can be compelled to testify, even over the defendant's objection. Accordingly, answer choice A is incorrect. Answer choice C is incorrect because, in federal court and a majority of states, the defendant cannot prevent the wife from testifying if she chooses to do so. Answer choice D is incorrect because, as explained with regard to answer choice B, the wife can be compelled to testify.

Question 5935 The police suspected a man was cooking methamphetamine in his home basement. From the street, the officers could detect no illicit activity in the basement, but they did see a beam of light in his side yard indicating a low window. The officers, who did not have a warrant, stepped over a low fence to enter his neighbor's yard and walked about 30 feet away from the neighbor's house. From this vantage, the officers saw evidence of methamphetamine production through a small unobstructed window into the basement. On the basis of this information, the officers obtained a search warrant and seized a large amount of methamphetamine from the man's home. The man was charged with drug offenses related to methamphetamine. The man has moved to suppress the evidence seized under the authority of the warrant, claiming that the evidence was obtained through an unconstitutional search in violation of the Fourth Amendment. Should the evidence be suppressed? Answers: No, because the officers were not within the curtilage of the neighbor's house when they saw evidence of methamphetamine production in the man's basement. No, because the man lacked a reasonable expectation of privacy in activities that he did not hide from his neighbors. Yes, under the fruit of the poisonous tree doctrine. Yes, because the plain view exception does not apply when a police officer is not legitimately on a premises.

Answer choice B is correct. The man's neighbors had a clear and unobstructed view into the man's basement through the small side window. Therefore, the man cannot successfully argue that he had a reasonable expectation of privacy in the activities or property in his basement. Although the officers presumably did trespass on his neighbor's property, the man has no standing to complain of their illegal conduct. Answer choice A is incorrect. Although whether the officers were within the curtilage of the neighbor's house when they saw evidence of the methamphetamine production would be relevant if the production was taking place in the neighbor's basement, it is not relevant to the issue of the constitutionality of the officer's visual search of the man's basement through its window. The determinative issue here is not the man's expectation of privacy in where the officers were standing, but in where they observed illegal activity. Answer choice C is incorrect. The exclusionary rule can apply not only to evidence initially seized as a result of the primary government illegality, but also to secondary "derivative evidence" discovered as a result of the primary taint, also known as the "fruit of the poisonous tree." In this case, the information acquired by the officers while on the neighbor's property was not acquired in violation of the man's constitutional rights because he did not have a reasonable expectation of privacy in this information. Therefore, there was no "poisonous tree" from which to produce an inadmissible "fruit." Answer choice D is incorrect because this limitation on the plain view rule applies when a police officer is illegitimately on the premises that the officer is searching. It does not apply to a trespass by the officer on the property of another individual that is not being searched.

Question 7394 A state raised its minimum wage to $15 per hour. A federal post office in the state pays some of its employees the federal minimum wage of $7.25 an hour. Can the post office be prosecuted for failing to adhere to the new state minimum wage? Answers: No, because the state minimum wage is preempted by federal law. No, because the state cannot regulate the federal government absent congressional consent. Yes, because the state minimum wage is not a tax on the federal government. Yes, because the post office at which the federal employees are employed is located in the state.

Answer choice B is correct. The states have no power to regulate the federal government unless Congress permits the state regulation or unless the state regulation is not inconsistent with existing federal policy. Here, the state's minimum wage statute penalizes the behavior of an agency of the federal government, which is immune from that regulation under the concept of sovereign immunity. Answer choice A is incorrect. When federal and state governments legislate in the same area, the Supremacy Clause provides that federal law supersedes conflicting state law. However, there is no conflict here that will trigger preemption. A state law conflicts with federal law if it allows conduct that is forbidden by the federal law or makes it impossible (or nearly so) to comply with both. Here, an employer can easily comply with both the state and federal law when it obeys the state law. Therefore, preemption does not apply. Answer choice C is incorrect because the states have no power to regulate the federal government unless Congress permits the state regulation or unless the state regulation is not inconsistent with existing federal policy. This immunity is not limited to state taxation, and even some state taxation may be permitted. Answer choice D is incorrect. Even though the post office employees are working within the state, the employees are working for an agency of the federal government. Although the employees' wages can be subject to state income tax, the federal government as an employer is not subject to the state's minimum wage.

Question 6353 The police suspected that a motorcycle gang was involved in dealing illegal firearms. After one week of surveilling the gang's clubhouse, the police obtained a warrant to search the entire clubhouse for any firearms. Without knocking and announcing their presence, the police broke down the front door of the clubhouse, spread out, and looked around. The police found various firearms in the house, both in plain view and in locked containers located throughout the clubhouse. They seized all of the firearms and arrested the gang's leader, who owned the clubhouse. Prior to trial, the gang leader moved to suppress the firearms found in the house. Should the court grant the gang leader's motion to suppress? Answers: Yes, because the police did not knock and announce their presence. Yes, because the police broke into locked containers in the clubhouse. No, because the police's failure to knock and announce does not require exclusion of the firearms. No, but only as to firearms found in plain view.

Answer choice C is correct. The interests protected by the knock-and-announce requirement do not include the shielding of potential evidence from discovery. Thus, violation of the knock-and-announce rule does not trigger the exclusionary rule with respect to evidence discovered as a result of a search conducted in violation of the knock-and-announce rule because the interests protected by the knock-and-announce requirement do not include the shielding of potential evidence from discovery. Therefore, although the police did not knock and announce their entry, the firearms are still admissible. Thus, answer choice A is incorrect. Answer choice B is incorrect. The search warrant permitted the officers to search the entire clubhouse for any firearms. This implicitly included the search of any containers that could hold firearms, including both locked and unlocked containers. Answer choice D is incorrect. The police had a valid warrant to search for firearms. Consequently, they were not limited to firearms found in plain view but could search the premises for wherever the firearms could be found, which includes within a locked container that is large enough to hold the firearms. Therefore, answer choice C is a more precise explanation for the admissibility of the evidence and is the better answer choice.

Question 3085 A woman went to visit her elderly grandfather, who suffered from severe dementia and numerous physical ailments. The grandfather had great difficulty walking, and was largely confined to a wheelchair. The woman had always been scared of her grandfather because he had always had a short temper and a violent streak. At one point during their visit, the grandfather became confused and seemed to think that the woman was a girlfriend from his youth. The grandfather told the woman that he knew she had cheated on him, and that he ought to get the gun out of his dresser and shoot her. The woman quickly ran to the dresser and grabbed the gun. The grandfather threatened that he would take the gun and shoot her, at which point the woman shot her grandfather, killing him instantly. The woman was charged with homicide offenses, and the woman claimed self-defense. The evidence at trial showed that she honestly believed her grandfather would kill her, although this belief was unreasonable given the grandfather's lack of mobility. Is the woman likely to be convicted of any homicide offense? Answers: The woman may be convicted of second-degree murder. The woman may be convicted of voluntary manslaughter. The woman may be convicted of involuntary manslaughter. The woman is unlikely to be convicted of homicide.

Answer choice B is correct. The woman would likely be convicted of voluntary manslaughter under the imperfect self-defense rule. Imperfect self-defense occurs when the person claiming self-defense unjustifiably kills her attacker, such as when she honestly but unreasonably believes self-defense is required. The rule reduces the charge from murder to voluntary manslaughter. Because the woman honestly but unreasonably believed in the need for self-defense, the imperfect self-defense rule would apply, and she would likely be convicted of voluntary manslaughter. Answer choice A is incorrect because the imperfect self-defense rule would reduce the charge to voluntary manslaughter. Answer choice C is incorrect because there was no involuntary manslaughter, or unintentional homicide committed with criminal negligence or during an unlawful act. Answer choice D is incorrect because the woman would be held liable because she was not entitled to use self-defense. Rather, under the imperfect self-defense rule, the charges would simply be reduced.

Question 5967 A college student was inducted into a secret society. After an oath ceremony, the active members told the inductee that they were planning to raid an animal lab and set all of the animals free. The inductee agreed to help and told the active members how noble their mission was. He encouraged them to bring as many people as possible to the animal lab, all the better to set more animals free. The active members and the inductee agreed to meet at the animal lab at midnight. At 11:00 p.m., the inductee decided not to join the secret society. He did not go to the animal lab, and he did not communicate his decision to the active members. He decided that they would know he had changed his mind when he did not show up. As planned, the active members broke into the animal lab at midnight and set the animals free. The next day, the police arrested the inductee for burglary. They told him that the active members had turned him in. In a state that has expanded burglary by statute to include non-dwellings, will the inductee likely be convicted of burglary? Answers: Yes, because he knew that the active members planned to break into the animal lab. Yes, because he did nothing to stop the active members from carrying out the crime. No, because he did not break into the animal lab to set all of the animals free. No, because he decided not join the secret society.

Answer choice B is correct. To legally withdraw (and therefore avoid liability for the substantive crime), an accomplice must (i) repudiate prior aid, (ii) do all that is possible to countermand prior assistance, and (iii) do so before the chain of events is in motion and unstoppable. A mere change of heart, a flight from the crime scene, an arrest by law enforcement, or an uncommunicated decision to withdraw is ineffective. Here, although the inductee changed his mind, he did not repudiate his words of encouragement or do anything to stop the break-in from occurring. An hour before the break-in was scheduled, the inductee simply had a change of heart and did not show up at the animal lab. He did not countermand his prior assistance by notifying the authorities or repudiating his prior aid by letting the active members know that he was not going to go to the animal lab. He also withdrew an hour before the break-in was scheduled, so most likely the chain of events was in motion and unstoppable at that point. Therefore, the inductee's attempted withdrawal was ineffective and he will likely be convicted of burglary as an accomplice. Answer choice A is incorrect because it misstates the rule. Mere knowledge that another person intends to commit a crime is not enough to make a person an accomplice. Moreover, this answer choice also misses the central issue, which is that the inductee did not effectively withdraw from the crime. Answer choice C is incorrect. The inductee did not effectively withdraw from the crime as discussed above. Thus, he is still liable for the burglary as an accomplice, even though he did not break into the animal lab. Answer choice D is incorrect because the inductee did not have to be an official member of the society to commit a crime with them, and his decision not to join is not enough to constitute withdrawal.

Question 6527 A man who had just purchased a new home was expecting his out-of-town cousin to come visit him for a few days. The cousin was arriving in the evening, while the man would still be at work, so the man told his cousin his address and that he would leave his door unlocked for him. The cousin took a taxi from the airport, and the taxi driver accidentally dropped the cousin off in front of the wrong house. The cousin opened the door of the incorrect house, which was also unlocked, and waited in the living room for the man to come home. A few minutes later, the owner of the house arrived and discovered the cousin sitting on his couch in his living room. If the owner sues the cousin for damages based upon trespass, will he succeed? Answers: No, because the cousin believed that the home belonged to the man. No, because the owner did not suffer any actual harm. Yes, because the cousin intentionally entered the owner's home. Yes, because the cousin unreasonably interfered with the owner's use of his property.

Answer choice C is correct. A trespass occurs when the defendant's intentional act causes a physical invasion of another's land. The defendant need only have the intent to enter the land, not the intent to commit a wrongful trespass. Here, the cousin intentionally entered the owner's house. Therefore, he is liable for trespass. Answer choice A is incorrect because the cousin is liable for trespass, even if he mistakenly believed that the owner's home was the man's home. Mistake of fact is not a defense. Answer choice B is incorrect because the cousin will still be liable for nominal damages due to his wrongful conduct, even if no actual harm occurred. Answer choice D is incorrect because it states the legal standard for a private nuisance, not trespass. Therefore, this answer choice is not determinative of the cousin's liability for trespass.

Question 416 A father was suspected by police of felony child abuse. This crime is punishable by a maximum of 20 years imprisonment. While in police custody and subject to police questioning, which was effected without coercion, the father made incriminating statements about the abuse suffered by his child, but he stopped short of confessing. Immediately thereafter, the police gave the father Miranda warnings which they had purposefully withheld until that time for strategic reasons. The father, reasonably concluding that he had already waived his right to remain silent by making the incriminating statements, made a confession. Subsequently, the father requested to speak with a lawyer. The prosecution seeks to introduce the father's confession at trial over the objection of the father's attorney. Should the court permit the prosecution to introduce the confession? Answers: Yes, because the confession was made after the father received Miranda warnings. Yes, because the father, if convicted, would not be subject to the death penalty. No, because the police tactic of questioning before giving Miranda warnings effectively circumvents the purpose of Miranda. No, because the confession violated the father's Sixth Amendment right to counsel.

Answer choice C is correct. Although the giving of Miranda warnings generally dissipates the taint of a prior Miranda violation with respect to a subsequent confession, this is not always the case. Here, the purpose of Miranda was circumvented by the police's intentional decision to initially withhold Miranda warnings in a circumstance in which these warnings would otherwise be required, as evidenced by the suspect's reasonable conclusion that his prior incriminating statements constituted a waiver of his right to remain silent. For this reason, answer choice A is incorrect. Answer choice B is incorrect because the exclusion of evidence in violation of Miranda does not turn on the type of punishment that can be imposed for the charged crime. Answer choice D is incorrect because the Sixth Amendment right to counsel is not generally triggered until the suspect has been charged with a crime.

Question 8426 In building a condominium on land adjacent to a lake, a contractor drove piles into the ground to provide a stable base on which to construct the condominium. Although the contractor used reasonable care in driving the piles, the vibrations from doing so caused cracks and other major damages to a nearby building. It was foreseeable that driving the piles, which is not an activity commonly engaged in, created a highly significant risk of physical harm, but there was no physical invasion of objects or particles as a result of driving the piles. Can the owner of the nearby building recover from the contractor for the damage done to the building under a theory of strict liability? Answers: No, because the contractor used reasonable care in driving the piles. No, because there was no physical invasion of objects or particles as a result of driving the piles. Yes, because driving piles is an abnormally dangerous activity. Yes, because of the res ipsa loquitur doctrine.

Answer choice C is correct. An activity is abnormally dangerous if that activity (i) creates a foreseeable and highly significant risk of physical harm even when reasonable care is exercised, and (ii) the activity is not commonly engaged in. Here, those requirements are met—the facts indicate both that driving piles is not an activity that is commonly engaged in and it creates a foreseeable and highly significant risk of physical harm. Thus, driving piles is an abnormally dangerous activity. Strict liability for an abnormally dangerous activity is available if the harm that actually occurs results from the risk that made the activity abnormally dangerous in the first place. In this case, the owner of the nearby building can recover for the damage to the building done by the vibrations because the risk of vibrations is what makes pile driving abnormally dangerous. Answer choice A is incorrect because strict liability may be pursued as a cause of action even though the defendant exercised reasonable care in conducting an abnormally dangerous activity. Answer choice B is incorrect because, unlike trespass to land, strict liability for an abnormally dangerous activity does not require a physical invasion of the affected land. Answer choice D is incorrect. The res ipsa loquitur doctrine permits the trier of fact in a negligence action to infer the existence of the defendant's negligent conduct in the absence of direct evidence of such negligence. Here, the building owner has pursued a strict liability action, under which the issue of the defendant's negligence is irrelevant.

Question 7473 The owner of a ranch devised the ranch as follows: "to my wife for life, and then to my grandchildren." The owner devised the remainder of his property to his only child. The owner was survived by his wife and only child. At the time of the wife's death, the owner's child did not have any children. The applicable jurisdiction adheres to the common law Rule Against Perpetuities. What is the status of the title to the ranch immediately after the wife's death? Answers: The child owns the ranch in fee simple absolute, because the contingent remainder in the grandchildren failed to vest when it became a possessory interest. The child owns the ranch in fee simple absolute, because of the Rule Against Perpetuities. The child owns the ranch in fee simple subject to an executory interest. The child owns the ranch in fee simple subject to a contingent remainder.

Answer choice C is correct. At the time of the owner's death, the wife had a life estate in the ranch, the owner's grandchildren held a contingent remainder, and the owner's child held the reversionary interest as the devisee of the owner's property other than the ranch. The grandchildren's interest was a remainder because it was a future interest created in a grantee that was capable of becoming an estate that is presently possessory upon the natural expiration of a prior possessory estate (i.e., the wife's life estate) that was created in the same conveyance in which the remainder was created. This interest was a contingent remainder because it was created in a grantee who was unascertainable. The owner's estate held a reversion in the ranch because there was the possibility that there would be no grandchildren when the wife died. Upon the wife's death, there was no longer a prior possessory estate of known fixed duration, so the grandchildren's interest was converted into an executory interest. An executory interest is a future interest in a third party that is not a remainder and that generally cuts the prior estate short upon the occurrence of a specified condition. In this case, it will terminate the interest held by the owner's child. Upon the wife's death, the interest of the owner's child was converted from a reversion, which is a future interest, to a present possessory interest. As this interest is only subject to the grandchildren's executory interest, the interest of the owner's child is a fee simple subject to an executory interest. Answer choice A is incorrect. Although at common law contingent remainders that did not vest by the time the preceding estate terminated were destroyed, most states have abolished this doctrine. Answer choice B is incorrect. The Rule Against Perpetuities does not apply because the owner's child is a measuring life. At the child's death, it will be known whether the child has any children. Thus, the contingent remainder in the child's children (i.e., the owner's grandchildren) will either vest or will fail by the end of a life in being plus 21 years. Answer choice D is incorrect because, although initially the interest of the owner's grandchildren was a contingent remainder, it was converted into an executory interest upon the death of the owner's wife.

Question 8355 A business learned that a former executive had accepted employment with a competitor in breach of a noncompete covenant in his contract with the business, likely a violation of state law. In federal district court, the business filed a complaint seeking an injunction against the former executive to enforce his obligations under the covenant. The former executive filed a counterclaim against the business based on an alleged violation of the federal Americans with Disabilities Act. There is no diversity between the parties. Prior to trial, the court granted a summary judgment motion filed by the business, and it dismissed the counterclaim. Immediately thereafter, the court, acting sua sponte, dismissed the action on the grounds that the court lacked subject matter jurisdiction. Is the court's dismissal of this action proper? Answers: No, because no party raised the issue of lack of subject matter jurisdiction. No, because the defendant's counterclaim raised a federal question issue. Yes, because the complaint filed by the business was based on a breach-of-contract claim. Yes, because the court dismissed the federal-law-based counterclaim.

Answer choice C is correct. For a federal district court to have subject matter jurisdiction based on a federal question, the action must arise under the Constitution, laws, or treaties of the United States. Here, the business sought to enforce, through an injunction, the noncompete covenant in its contract with the former executive. This complaint is based on contract law, and it therefore is not based on a federal question. Consequently, the dismissal of this action was proper. Answer choice A is incorrect because, while a party may seek to dismiss an action based on the court's lack of subject matter jurisdiction, the court may also do so of its own accord. Answer choice B is incorrect because in considering whether federal-question jurisdiction exists with respect to an action, the sole focus is on the complaint. Therefore, even though the defendant's counterclaim did raise a federal question, the court lacked federal-question subject matter jurisdiction because the complaint was based on a state cause of action, not a federal cause of action. Answer choice D is incorrect because a counterclaim is not considered in determining the existence of federal-question jurisdiction. Thus, the court's dismissal of the action brought by the business was proper, regardless of its ruling on the defendant's counterclaim.

Question 6209 A man seeking to cover his bald spot purchased a toupee from a reputable hairpiece company. Unfortunately, the toupee would not stay in place when the man played racquetball at his country club, despite the fact that he correctly followed the company's instructions regarding how to keep the toupee in place. The man sued the company, but after a bench trial, the judge ruled in favor of the hairpiece company because the company included a valid, legal disclaimer that the toupee might not stay in place during physical activities, including sports such as racquetball. The man's weekly racquetball partner then sued the company because his hairpiece also did not stay in place when he played racquetball. The man and his racquetball partner discussed the man's case in detail, and the man offered his racquetball partner some strategic advice regarding how the racquetball partner could present his legal case, including demanding a jury trial. At the jury trial, the only issue was whether the company's disclaimer was valid. The company filed a motion to dismiss the racquetball partner's action based on collateral estoppel because this issue had been decided in the previous case brought by the man. For whom should the court rule? Answers: The company, because the issue sought to be precluded was the same as that litigated in the prior action. The company, because the man and his racquetball partner were in privity. The racquetball partner, because the prior action was not brought by him. The racquetball partner, because the prior action was not tried before a jury.

Answer choice C is correct. The doctrine of issue preclusion, often called "collateral estoppel," precludes the re-litigation of issues of fact or law that have already been necessarily determined by a judge or jury as part of an earlier claim. Unlike claim preclusion, issue preclusion does not require strict mutuality of parties. Issue preclusion requires that the party against whom the issue is to be precluded (or one in privity with that party) must have been a party to the original action. Here, because the racquetball partner was not a party to the first action brought by the man, nor was he in privity with the man, issue preclusion does not apply with regard to the issue of the validity of the company's disclaimer. Answer choice A is incorrect because, although the issue of the validity of the disclaimer was litigated in the prior action, the racquetball partner was not a party to that action, nor is the partner in privity with the man who was a party to that action. Accordingly, collateral estoppel cannot be used against him. Answer choice B is incorrect because, although the man and his racquetball partner discussed their cases and the man offered his racquetball partner advice about how to litigate the matter, the two are not in privity, which exists when there is some legal relationship between two persons. Answer choice D is incorrect. Although issue preclusion applies only when the issue has been actually litigated in the prior action, it does not require the issue to have been decided by a jury rather than a judge.

Question 8518 A fan ordered a sweatshirt with the logo of a local sports team from the team's website. In a box labeled "Comments," the fan, who planned on wearing the sweatshirt on game days, wrote, "blue." Via email, the team sent a notice acknowledging the order, but reserving the right to send a sweatshirt in either of the team's colors: red or blue. The team shipped the fan a red sweatshirt. Which of the following arguments would not support the fan's position that she does not have to pay for the sweatshirt? Answers: Because the fan was not a merchant, the retailer's additional term in the acknowledgment is not part of contract. The fan did not separately agree to receive a red sweatshirt. The team's acceptance did not mirror the fan's offer. The team's shipment included nonconforming goods.

Answer choice C is correct. The mirror image rule, which states that the acceptance must mirror the terms of the offer, does not apply to a sale of goods, such as a sweatshirt. Answer choices A, B, and D are incorrect because they all support the fan's position that she does not have to pay for the sweatshirt. Answer choice A is incorrect because, if the acknowledgment is an acceptance, since the fan is not a merchant, a different term (i.e., a sweatshirt of either blue or red) is not part of the contract. Answer choice B is incorrect. Because the fan is not a merchant, any additional terms in the acceptance are treated as a proposal. This proposal must be separately accepted by the fan to become part of the contract. Answer choice D is incorrect because the failure of the team to ship conforming goods (i.e., a blue sweatshirt) would constitute a breach of the team's contract obligations.

Question 7436 Occupying the field of alien registration, federal law imposes a series of requirements on the registration of aliens, including the requirement that aliens who remain in the United States for more than 30 days must register with the federal government and carry proof of registration. Under federal law, willful failure by an alien to register or carry proof of registration is punishable as a misdemeanor. A recently enacted state statute also makes the willful failure of an alien to register with the federal government or carry proof of registration punishable as a misdemeanor. The federal law does not contain an express preemption of state laws criminalizing the failure of aliens to register or carry proof of registration, nor does it contain a "savings clause" that explicitly allows state laws that regulate the registration of aliens. The state statute has been challenged in federal district court as invalid under the Supremacy Clause of Article VI of the United States Constitution. Is the state statute valid under the Supremacy Clause? Answers: Yes, because the state statute parallels the federal law by punishing the same conduct and to the same degree as the federal law. Yes, because federal law does not expressly preempt state laws criminalizing the failure of aliens to register or carry proof registration. No, because the federal law occupies the entire field of alien registration. No, because the federal law does not contain a savings clause.

Answer choice C is correct. When Congress intends for a federal law to occupy the field, such as is the case with regard to the registration of aliens in the United States, federal law preempts state law dealing with the same subject even if federal law does not expressly preempt state law with regard to that subject. Answer choice A is incorrect because implied preemption of state law based on federal occupation of the field reflects a congressional decision to foreclose any state regulation in the area, even if it is parallel to federal standards. Answer choice B is incorrect because even if federal law does not expressly preempt state law with regard to a certain subject, federal law can impliedly preempt state law when Congress intended for federal law to occupy the field. Answer choice D is incorrect because, while the existence of a saving clause that explicitly allows state law with regard to a subject prevents federal preemption, the absence of such a clause does not by itself establish federal preemption. Instead, a court must examine federal law to determine whether it expressly or impliedly preempts state law.

Question 8555 A nanny and a mother entered into negotiations for the nanny to provide care for the mother's child. After several emails back and forth, the mother and the nanny agreed in writing that the nanny would care for the child from 9:00 a.m. to 5:30 p.m., Monday through Friday, for $750 per week. The nanny provided care for the child as specified for a week. The mother found that, to arrive at her job on time in the mornings, she needed the nanny to begin child care at 8:45 a.m. instead and emailed the nanny to that effect. By a reply email, the nanny agreed to the earlier start time. After several weeks during which time the nanny often failed to arrive before 9:00 a.m., the mother informed the nanny that the mother had found a replacement and that the nanny's services would no longer be required. It took the nanny several months to find another nanny position. She sued the mother for the income she would have earned under their agreement during the time she was unemployed. The mother offers into evidence the nanny's agreement to begin working at 8:45 a.m. each day. The nanny objects. Is the court likely to admit this evidence? Answers: No, because the agreement as to the earlier start time arose after the contract was executed. No, because the contract between the parties did not include a requirement for the nanny to begin working at 8:45 a.m. Yes, because the parol evidence rule does not apply. Yes, because there was no new consideration for the earlier start time.

Answer choice C is correct. While the parol evidence rule generally prevents a party to a written contract from presenting prior or contemporaneous extrinsic evidence that contradicts or is inconsistent with the terms of the contract as written, it does not apply to subsequent agreements like the email agreement regarding the earlier start time. Answer choice A is incorrect because the parol evidence rule does not apply to agreements subsequent to the execution of a contract. Answer choice B is incorrect because, even though the original contract did not include a requirement that the nanny begin working at 8:45 a.m., the parties subsequently agreed that the nanny would do so. Answer choice D is incorrect because, while it is true that the court might refuse to enforce the earlier start time requirement due to the lack of additional consideration, this question asks about whether evidence of the agreement would be admitted, not whether it would be enforced.

Question 7557 A retailer sponsored a game that required individuals to collect game pieces in order to win various monetary prizes. An employee of the retailer embezzled the rare game pieces necessary for the significant monetary prizes. The employee then separately contacted various individuals, each of whom was given a rare game piece. The employee did not disclose and the individuals did not learn that the employee gave game pieces to anyone else. Each individual then submitted the necessary game pieces, including the rare game piece provided by the employee, to the retailer for a monetary prize. In order to obtain a prize, each individual falsely stated that he had obtained all of the game pieces through authorized, legitimate channels. Upon receipt of the prize money, each individual paid 50 percent of the money to the employee, as previously agreed. Among other offenses, the employee and all of the individuals were charged with entering into a single conspiracy to obtain money from the retailer by false pretenses. Can the defendants properly be convicted of this conspiracy? Answers: Yes, because each individual falsely stated that he had obtained all of the necessary game pieces through authorized, legitimate channels. Yes, because each individual shared 50 percent of the prize received from the retailer with the employee. No, because only the employee embezzled the games pieces from the retailer. No, because the individuals did not know of the participation of the other individuals in the scheme.

Answer choice D is correct. At common law, conspiracy is (i) an agreement (ii) between two or more persons (iii) to accomplish an unlawful purpose (iv) with the intent to accomplish that purpose. Here, the unlawful purpose was to obtain money from the retailer by false pretenses. False pretenses is (i) obtaining title to the property (ii) of another person (iii) through the reliance of that person (iv) on a known false representation of a material past or present fact, and (v) the representation is made with the intent to defraud. In this case, each individual and the employee had the intent to obtain money from the retailer through the retailer's reliance on each individual's false representation that he had obtained all of the game pieces through authorized, legitimate channels. However, there was not an agreement among all of the individuals to participate in the employee's scheme, but only separate agreements between the employee and each individual to obtain money from the retailer by false pretenses. Consequently, the employee and all of the individuals cannot properly be convicted of entering into a single conspiracy. Answer choice A is incorrect. Although each individual satisfied the requirements for the crime of obtaining money by false pretenses, there was not an agreement among all of the individuals to participate in the employee's scheme, but only separate agreements between the employee and each individual to obtain money by false pretenses. Answer choice B is incorrect. Although the fact that each individual shared 50 percent of the prize received with the employee demonstrates the existence and furtherance of a conspiracy, that conspiracy only existed between the employee and each individual, not between the employee and all of the individuals in a single conspiracy. Answer choice C is incorrect because each person in a conspiracy need not engage in every illegal act necessary for the accomplishment of the purpose of the conspiracy.

Question 4359 A city adopted an ordinance that prohibited any apartment building from housing more than one convicted felon on the premises. A convicted felon owned an apartment building in the city. He lived in one of the apartments in the building with his brother, who was also a convicted felon. The building owner received a citation from the city for violating the ordinance, and was ordered to pay a fine and bring the building into compliance. The building owner sued the city, arguing that the ordinance violated his constitutional rights. What is the best argument for striking down the ordinance? Answers: The ordinance violates the Privileges and Immunities Clause. The ordinance violates the Equal Protection Clause. The ordinance violates the Eighth Amendment. The ordinance violates the Due Process Clause.

Answer choice D is correct. The guarantee of substantive due process is based upon the idea that laws should be reasonable and not arbitrary, and ensures that a governmental action that infringes upon a fundamental right is generally subject to strict scrutiny. Related persons have a fundamental right to live together in a single household. In this case, the ordinance prevented the man from living with his brother. Thus, the man's best argument is that the ordinance, by prohibiting him from living with a family member, violates substantive due process. Answer choice A is incorrect because the Privileges and Immunities Clause, which prohibits a state from discriminating against the citizens of another state, is not implicated in this case. Answer choice B is incorrect because the law does not employ a suspect classification, and thus any equal protection claim would be subject to rational basis review. The man would have a difficult time showing that the ordinance did not satisfy this standard. Answer choice C is incorrect because state and local governments do not violate the Eighth Amendment simply by imposing special disabilities on convicted felons, especially when doing so is reasonably related to protecting public safety.

Question 7478 Driving home from a party, a guest was involved in an accident with a bicyclist. The cyclist sued both the guest and the host of the party. The guest settled with the cyclist before trial, but the host did not. At trial, the host called the guest as a witness. The host expected the guest to testify, as he had done at his deposition, that he was sober at the time of the accident. However, the guest testified that he was intoxicated at the time of the accident. The host confronted the guest with his deposition testimony, but the guest denied making the statement. The host then sought to introduce the guest's deposition statement as substantive evidence that the guest was sober at the time of the accident. The cyclist objects that the statement is inadmissible hearsay. For whom should the court rule? Answers: For the cyclist, because the guest is no longer a party to the action. For the cyclist, because the guest's prior inconsistent statement was not made at a trial or hearing over which a judge presided. For the host, because the deposition statement calls into question the guest's veracity. For the host, because the guest is testifying at the trial and can be cross-examined about the statement.

Answer choice D is correct. The guest's deposition statement, although it is being offer for the truth of matter asserted and therefore would otherwise be hearsay, is nonhearsay because it is a prior inconsistent statement made, under oath at a prior proceeding, by the witness who is on the stand and can be cross-examined about it. Answer choice A is incorrect because, unlike an opposing party's statement, a prior inconsistent statement can be admissible as substantive evidence even though the declarant-witness is not a party to the action. Answer choice B is incorrect because, although a prior inconsistent statement made by the declarant-witness at a prior trial or a hearing may be introduced as substantive evidence in the current trial, such a prior inconsistent statement made at a deposition may also be introduced as substantive evidence. Statements made by a deponent are made under oath. Answer choice C is incorrect because, while the deposition statement does call into question the guest's veracity and so may also be used for the purpose of impeaching the guest's testimony at trial, the deposition statement is admissible as substantive evidence because it a prior inconsistent statement made under oath by the deponent-witness.

Question 409 Two police officers stopped a car for a minor traffic violation. While one of the officers dealt with the driver and the traffic violation, the other officer talked with the passenger. Seeking to question the passenger about gang involvement, but lacking reasonable suspicions of criminal activity, the officer ordered the passenger out of the car. As the passenger exited the car, the officer saw a bulge in the passenger's coat which the officer suspected might be a gun. Upon patting down the passenger, the officer felt the handle of a revolver and removed the gun. The passenger, who was a convicted felon, was charged with the possession of a gun by a prohibited possessor. Prior to trial, the defendant sought to suppress the gun as evidence, contending that its seizure was unconstitutional. Should the court suppress the gun as evidence? Answers: Yes, because, since the car had been stopped for a traffic violation, the officer could not order the passenger, who had not committed the violation, to exit the car. Yes, because, since the officer lacked reasonable suspicion that the passenger was engaged in criminal activity, the officer could not pat down the passenger. No, because a valid traffic stop gives an officer the right to pat down a passenger. No, because the gun was discovered as a consequence of a valid Terry stop and frisk.

Answer choice D is correct. The traffic violation gave the police officers a valid reason for stopping the car. As part of the stop, the police officer could order the passenger to exit the vehicle. As the passenger did so, the police officer gained a reasonable suspicion that the passenger was armed and dangerous. Consequently, the officer could pat down the passenger for weapons. Answer choice A is incorrect because the police may order a passenger to exit a car that is legitimately stopped for a traffic violation, even though the passenger is not responsible for the traffic violation. Compelling the passenger to exit the car constitutes a de minimis additional intrusion. Answer choice B is incorrect because the officer's justification for ordering the passenger to exit the car arose from the traffic violation. The fact that the officer also had another reason for doing so that was not supported by a reasonable suspicion as constitutionally required does not prevent the officer from ordering the passenger to exit the car. Answer choice C is incorrect because an officer may not conduct a pat-down as a consequence of a valid traffic stop. The officer must have a reasonable suspicion that the passenger is armed and dangerous.

Question 7230 A student sued his college professor for battery. The student testified at trial about the full incident, explaining that the professor had pushed him against a wall while they were involved in a heated discussion when he visited the professor's office during his scheduled office hours. He also explained that because he had other assignments and exams that week, he waited to go to the doctor, despite being in significant pain, until the day after his political science exam. When the student's attorney asked him the date he went to the doctor, he said he could not remember the exact date. The student's attorney then offered the student his assignment planner to refresh his memory. Reading from the planner, the student testified, "My planner states 'Poly sci. test today' on the 13 and 'Go to the doctor' on the 14." The professor moves to strike the student's testimony from the record. Should the court strike the student's testimony? Answers: No, because it was permitted under the doctrine of past recollection recorded. No, because it was permitted under the doctrine of present recollection refreshed. Yes, because the planner was not offered into evidence by the defense. Yes, because the student did not testify based on his refreshed recollection.

Answer choice D is correct. Under the doctrine of present recollection refreshed, a witness may examine any item (e.g., writing, photograph) to "refresh" the witness's present recollection. The witness's testimony must be based on the witness's refreshed recollection, not on the item itself (e.g., the witness cannot read from the refreshing document). Here, the student's attorney offered the student his assignment planner to refresh his memory. After looking at the planner, the student read directly from the planner, rather than testifying based on his refreshed recollection. Accordingly, the court should strike the student's testimony. Answer choice A is incorrect. If a witness is unable to testify about a matter for which a record exists, that record is not excluded as hearsay if the following foundation is established: (i) the record is on a matter that the witness once knew about, (ii) the record was made or adopted by the witness when the matter was fresh in the witness's memory, (iii) the record accurately reflects the witness's knowledge, and (iv) the witness states that she cannot recall the event well enough to testify fully and accurately, even after consulting the record on the stand. In this case, it is not certain whether these requirements have been met as the student did not indicate that he could not testify even after consulting the planner, but rather read from the planner shortly after being handed it. Moreover, the facts do not indicate that the other requirements have been met. Answer choice B is incorrect. Under the doctrine of present recollection refreshed, the witness's testimony must be based on the witness's refreshed recollection, not on the item itself (e.g., the witness cannot read from the refreshing document). Thus, the doctrine of present recollection refreshed does not apply here because the student did not testify based on his refreshed recollection, but instead read directly from the planner. Answer choice C is incorrect. Under the recorded recollection exception to the hearsay rule, the record, if admitted, may be read into evidence, but it may be received as an exhibit only if offered by an adverse party. Here, if the defense had offered the planner into evidence, it could be read or offered as an exhibit. However, the planner must first meet the requirements for admission before it can be read into evidence or offered as an exhibit. As discussed above, the requirements for the recorded recollection exception have not been met and thus the planner cannot be read into evidence, regardless of whether the defense offered it.


Set pelajaran terkait

Chapter 16: Music After Beethoven: Romanticism

View Set

Substance-Related and Addictive Disorders

View Set

Master Educator Final Review, Master Educator Final Reviewc.

View Set

Chapter 10 Weathering and Soil Formation

View Set

Chapter 1 Mental Health and Mental Illness

View Set

Exam 2, Principles of management Chap 7, 8, 9, 10

View Set